0% found this document useful (0 votes)
19 views21 pages

Abstract - Algebra - Home I

The document presents solutions to 9 problems involving abstract algebra. It includes proofs of various statements about integers, greatest common divisors, least common multiples, and properties of squares and sums of squares modulo certain numbers. Explicit representations of residue classes are also provided.
Copyright
© © All Rights Reserved
We take content rights seriously. If you suspect this is your content, claim it here.
Available Formats
Download as PDF, TXT or read online on Scribd
0% found this document useful (0 votes)
19 views21 pages

Abstract - Algebra - Home I

The document presents solutions to 9 problems involving abstract algebra. It includes proofs of various statements about integers, greatest common divisors, least common multiples, and properties of squares and sums of squares modulo certain numbers. Explicit representations of residue classes are also provided.
Copyright
© © All Rights Reserved
We take content rights seriously. If you suspect this is your content, claim it here.
Available Formats
Download as PDF, TXT or read online on Scribd
You are on page 1/ 21

Abstract Algebra 2024–I

Problem Set 01
Luis Arteaga, Jhon Duta, Santiago López, Edwin Hurtado
March 24, 2024

Problem 1. For each of the following pairs of integers a and b, determine their
greatest common divisor, their least common multiple, and write their greatest
common divisor in the form ax + by for some integers x and y.

a) a = 792, b = 275

b) a = 507885 , b = 60808

We denote the least common multiple of a and b as

a·b
[a.b] = .
(a, b)

Solution. For a = 792, b = 275

792 = 2(275) + 242


275 = 1(242) + 33
242 = 7(33) + 11
33 = 3(11) + 0
then, (792, 275) = 11.

Calculating the least common multiple gives

792 · 275
[792, 275] = = 29800,
(792, 275)

1
then

11 = 242 − 7(33)
= 242 − 7[275 − 242]
= 8(242) − 7(275)
= 8[792 − 2(275)] − 2(275)
= (8)792 + (−23)(275).

Then, 11 = 8(792) + (−23)(275). □


Solution. For a = 507885, b = 60808

507885 = 8(60808) + 21421


60808 = 2(21421) + 17966
21421 = 1(17966) + 3455
17966 = 5(3455) + 691
3455 = 5(691) + 0
then, (507885, 60808) = 691.

Calculating the least common multiple gives

507885 · 60808
[507885, 60808] = = 44693880,
(507885, 60808)

then,

691 = 17966 − 5(3455)


= 17966 − 5[21421 − 17966]
= 6(17966) − 5(21421)
= 6[60808 − 2(21421)] − 5(21421)
= 6(60808) − 17(21421)
= 6(60808) − 17[507885 − 8(60808)]
= 142(60808) − 17(507885).

Then, 691 = 142(60808) + (−17)(507885). □

Problem 2. Prove that if n is composite then there are integers a and b such that
n divides ab but n does not divide either a or b.

Proof. Let be n > 1 be compositive. By the fundamental theorem of Arithmetic,

2
we have that

n = a1 a2 , · · · , ak where ∀i ∈ {1, 2, · · · , k} : ai is a prime.

Let
a = a1 ∧ b = a2 , · · · , ak .

Then
1·n=n=a·b

where a, b > 1, and


n|ab.

Then, since a is a prime, we have that

n∤a

(because there does not exists x0 ∈ Z such that x0 · n = a, because a is a prime.)


Let’s show that by contradiction that n ∤ b. Let’s suppose that n|b, then there is an
integer k > 1, such that

b = kn
=⇒ ab = akn
=⇒ n = akn
=⇒ 1 = ak

But this follows a contradiction since a, k > 1. Then

n ∤ b.

Problem 3. If p is a prime, prove that there do not exist nonzero integers a and b

such that a2 = pb2 . (Why this proves p is not a rational number.)

Lemma 1. (Euclide’s Lemma) If p is a prime and p|ab, then p|a or p|b.

Proof. Let p > 1 prime. Let’s show it by Reduction ad Absurdum.


Let’s suppose that there exists a, b ∈ Z\{0} : a2 = pb2 . Let’s suppose without

3
lose of generality that a and b do not has factors in common, i.e.,

(a, b) = 1 (1)

By our assumption, we get


p | a2

Then, by (lemma 1) we get,


p | a, (2)

then
a = k0 p, k0 ∈ Z,

and
a2 = k02 p2 .

By our assumption we get that

k02 p2 = a2 = pb2 =⇒ pk02 = b2 =⇒ p | b2

By Euclid’s Lemma (1), we get


p | b. (3)

By (2) and (3)

p|a ∧ p | b ⇐⇒ a = k0 p ∧ b = k1 p, k0 , k1 ∈ Z.

Then, p is a common divisor of a and b, that follow a contradiction with (1).


Therefore , there do not exists a, b ∈ (Z\{0}) : a2 = pb2 .

Problem 4. Write down explicitly all the elements in the residue classes of Z/18Z.

¯
Solution. We have that for Z/18Z = {0̄, 1̄, 2̄, · · · , 17}.

4
Then

0̄ = {18k0 /k0 ∈ Z}
1̄ = {18k0 + 1/k0 ∈ Z}
2̄ = {18k0 + 2/k0 ∈ Z}
3̄ = {18k0 + 3/k0 ∈ Z}
4̄ = {18k0 + 4/k0 ∈ Z}
5̄ = {18k0 + 5/k0 ∈ Z}
6̄ = {18k0 + 6/k0 ∈ Z}
7̄ = {18k0 + 7/k0 ∈ Z}
8̄ = {18k0 + 8/k0 ∈ Z}
9̄ = {18k0 + 9/k0 ∈ Z}
¯ =
10 {18k0 + 10/k0 ∈ Z}
¯ =
11 {18k0 + 11/k0 ∈ Z}
¯ =
12 {18k0 + 12/k0 ∈ Z}
¯ =
13 {18k0 + 13/k0 ∈ Z}
¯ =
14 {18k0 + 14/k0 ∈ Z}
¯ =
15 {18k0 + 15/k0 ∈ Z}
¯ =
16 {18k0 + 16/k0 ∈ Z}
¯ =
17 {18k0 + 17/k0 ∈ Z}.

Problem 5. Suppose a = an 10n + an−1 10n−1 + · · · + a1 10 + a0 is any positive


integer. Show that a ≡ an + an−1 + · · · + a1 + a0 (mod9). (Note that this is the
usual arithmetic rule that the remainder after division by 9 is the same as the sum
of the decimal digits mod 9 . In particular, an integer is divisible by 9 if and only
if the sum of its digits is divisible by 9 ).

Proof. Let’s use the following Lemma.

Lemma 2. If a ≡ b mod n and c ≡ d mod n. Then

i) ac ≡ bd mod n.

ii) a + c ≡ b + d mod n.

5
Clearly,
1 ≡ 10 mod 9.

Then by lemma (2), we get

10i ≡ 1i = 1 mod 9.

Then for all 0 ≤ i ≤ n, then

ai 10i ≡ ai mod 9.

Then,

a = an 10n + an 10n−1 + · · · + a1 10 + a0 ≡ an + an−1 + · · · + a1 + a0 mod 9.

Clearly a is divisible by 9 if only if a ≡ 0 mod 9 and if only if

an + an−1 + · · · + a1 + a0 ≡ 0 mod 9.

Problem 6. Compute the remainder when 37100 is divided by 29 .

Solution. Let
372 = 1369 = 29(47) + 6 ≡ 6 mod 29.

Then

374 ≡ 62 = 36 ≡ 7 mod 29
378 ≡ 72 = 49 ≡ 20 mod 29
3716 ≡ 202 = 400 = 29(13) + 23 ≡ 29mod 29
3732 ≡ 232 = 29(18) + 7 ≡ 7 mod 29
3764 ≡ 72 ≡ 20 mod 29
3764+32+4 = 3764 3732 374
≡ 20 · 7 · 7
≡ (29) · 33 + 23
≡ 23 mod 29.

Then, the remainder of 37100 when it is divided by 29 is 23.

6
Problem 7. Prove that the squares of the elements in Z/4Z are just 0 and 1.

Proof.

Lemma 3. If a ≡ b mod n =⇒ ai ≡ bi mod n, ∀i ∈ N.


(We will use ⊻ to denote exclusive disjunction).
We have to prove that if a ∈ Z, then

a2 ≡ (0 ⊻ 1) mod 4.

Clearly
a ≡ r mod 4, 0 ≤ r < 4.

Then, by (3)

a2 ≡ r2 mod 4,

then

02 = 0 ≡ 0 mod 4,
12 = 1 ≡ 1 mod 4,
22 = 4 ≡ 0 mod 4,
32 = 9 ≡ 1 mod 4.

Then, a2 ≡ (0 ⊻ 1)mod 4.

Problem 8. Let a, b ∈ Z. Prove that a2 + b2 never leaves a remainder of 3 when


divided by 4 . (Hint: use the previous exercise.)

Proof. Let a, b ∈ Z.
Since a2 ≡ (0 ⊻ 1) mod 4 and b2 ≡ (0 ⊻ 1) mod 4 we have four cases:

a2 + b2 ≡ 0 + 0 = 0 mod 4,
2 2
a +b ≡ 1 + 0 = 1 mod 4,
2 2
a +b ≡ 0 + 1 = 1 mod 4,
a2 + b2 ≡ 1 + 1 = 2 mod 4.

7
Problem 9. Prove that the equation x2 + y 2 = 3z 2 has no solutions for x, y, z ∈
Z\{0}.

Proof. let’s show by contradiction, let’s suppose there exists a nonempty solution
set for the equation
x2 + y 2 = 3z 2 , (4)

say,
S = {(x, y, z)| x2 + y 2 = 3z 2 }.

Let
A = {x| (x, y, z) ∈ S and x ∈ Z+ }.

By the well ordering principle. A has a minimal element, say x0 . Then, let
(x0 , y0 , z0 ) be a solution.
Now using mod 4 and the results of the previous exercises, we have that

x20 + y02 ≡ (0, 1, or 2) mod 4, (5)

and z02 ≡ (0, 1) mod 4, and clearly 3z02 ≡ 3 mod 4.


But x20 y02 = 3z02 ≡ 3 mod 4, generates a contradiction.
So then
3z02 ≡ 0 mod 4 =⇒ z02 ≡ 0 mod 4.

By the last, 2 divides z0 . Similarly, since x20 + y02 ≡ 0 mod 4, that follows

x20 ≡ y02 ≡ 0 mod 4.

Then 2 divides x0 and y0 . We have a new solution


x0 y0 z0
( , , ),
2 2 2
for (4).
But x20 < x0 , follows a contradiction with minimality of x0 . Hence we found a
contradiction and then S = ∅ showing that (4) has not a solution.

Problem 10. Prove that if ā, b̄ ∈ (Z/nZ)× , then ā · b̄ ∈ (Z/nZ)× .

Proof. Lets recall that

(Z/nZ)× = {k̄ | k̄ ∈ (Z/nZ) : k̄ · c̄ = 1̄, where c̄ ∈ (Z/nZ)}. (6)

Let ā, b̄ ∈ (Z/nZ)× , then by (6), we have that ā, b̄ has an inverse x̄ and z̄ respectively.

8
Then

(ā · b̄)(z̄ · x̄) = ā · (b̄ · z̄)x̄


= ā1̄x̄
= āx̄
= 1̄.

Thus, ā · b̄ ∈ (Z/nZ)× .

Problem 11. Let n ∈ Z, n > 1, and let a ∈ Z with 1 ≤ a ≤ n. Prove


if a and n are not relatively prime, there exists an integer b with 1 ≤ b < n
such that ab ≡ 0(modn) and deduce that there cannot be an integer c such that
ac ≡ 1(modn).

Proof. Since (n, a) = k, k > 1, then


a n
= c and = b, b, c ∈ Z.
k k
Then
n a
a · b = a · ( ) = n · ( ) = n · c ≡ 0 mod n.
k k
By reduction ad absurdum, let’s suppose that there exists c ∈ Z, such that

ac ≡ 1 mod n
=⇒ acb ≡ b mod n
=⇒ (ab)c ≡ b mod n.

But

abc ≡ 0 mod n,

and
b ̸≡ 0 mod n.

Since, 1 ≤ b < n. Thus we found a contradiction.

Problem 12. Let n ∈ Z, n > 1, and let a ∈ Z with 1 ≤ a ≤ n. Prove that if a and
n are relatively prime then there is an integer c such that ac ≡ 1(modn). (Use the
fact that the g.c.d. of two integers is a Z-linear combination of the integers.)

Proof. Let (a, n) = 1.

9
Then

1 = ca + dn, c, d ∈ Z
=⇒ ca − 1 = dn, c, d ∈ Z
=⇒ ca ≡ 1 mod n.

Problem 13. Conclude from the previous two exercises that (Z/nZ)× is the set of
elements ā of Z/nZ with (a, n) = 1 and hence prove Proposition 4 . Verify this
directly in the case n = 12.

Proof. Let’s prove that


(Z/nZ)× = W, (7)

where
W = {ā ∈ Z/nZ | (a, n) = 1}.

(⇒) By definition, we know that

(Z/nZ)× = {ā ∈ Z/nZ | ā · c̄ = 1, for some c̄ ∈ Z/nZ}. (8)

Let n ∈ Z+ . We have by (11) that

¯ = 0̄.
n ∈ Z, n > 1, 1 ≤ a ≤ n : (a, n) > 1 ⇒ ∃b ∈ Z, 1 ≤ b < n : ab (9)

By contrapositive in (9) we get that

¯ ̸= 0̄ ⇒ (a, n) = 1.
n ∈ Z, n > 1, 1 ≤ a ≤ n : ∀b ∈ Z, 1 ≤ b < n : ab (10)

Let ā ∈ (Z/nZ)× , then by (8), there exists c̄ ∈ Z/nZ, such that ā · c̄ = 1.


Note that ā · c̄ = 1̄ implies that

¯ ̸= 0.
∀b ∈ Z, 1 ≤ b < n : ab (11)

Let’s show that (11) holds by contradiction. Let’s suppose that

¯ = 0̄.
∃b ∈ Z, 1 ≤ b < n : ab

10
Then
ā · c̄ · b̄ = c̄ · 0 = 0
⇒1 · b̄ = 0
⇒b̄ = 0.

But this is a contradicction since 1 ≤ b ≤ n. Then (11) holds.


Therefore, by (10) (a, n) = 1 and

(Z/nZ)× ⊆ W. (12)

(⇐) By exercise (12) we have that

n ∈ Z, n > 1, 1 ≤ a ≤ n : (a, n) = 1 ⇒ ∃c ∈ Z : ac ≡ 1 mod n. (13)

Let ā ∈ W, then (a, n) = 1.


Directly by (13) we have that, there exists c ∈ Z such that

ac ≡ 1 mod n,

i.e,
ā · c̄ = 1,

then ā ∈ (Z/nZ)× by (8), i.e,

W ⊆ (Z/nZ)× . (14)

Since (12) and (14) holds, (7) holds, we are done.

Problem 14. (a) Prove that if n is squarefree (i.e., n > 1 and n is not divisible by

the square of any prime), then n is irrational.

(b) Prove that 3 2 is irrational.

Proof. a)

Let’s suppose that n is rational, i.e.,
√ a
n= , a ∈ Z, b ∈ Z\{0}.
b

a
Without lose of generality, let’s suppose that b is in lower terms, i.e.,

(a, b) = 1. (15)

11
2
Also, n = ab2 =⇒ a2 = nb2 . Since n is squarefree we can rewrite it as a product
of primes, where n is not divisible by the square of any prime. Let p be a prime
divisor of n, so that
n = pq, q ∈ Z, p a prime

and
(p, q) = 1. (16)

Since
a2 = nb2 = p(qb2 ), (17)

then p|a2 . By Euclid’s Lemma(1), we obtain p|a.


Then
a = mp, m ∈ Z =⇒ a2 = m2 p2 . (18)

By (17), (18), we get that

m2 p2 = pqb2 =⇒ m2 p = qb2 =⇒ p|qb2 .

By (16), we have that p ∤ q, and by Euclid’s Lemma(1)

p|b2 .

Applying again Euclide’s Lemma (1), we get that p|b. But p|b and p|a implies that
p is a common divisor of a and b. That follows a contradiction with (15). Then,

n is not rational.

Proof. b) We will use the following proposition in the proof.

Lemma 4. Let a ∈ Z, then a3 is even if only of a is even.


3
Let’s suppose that 2 is rational, then

3 a
2= ; a ∈ Z, b ∈ Z\{0},
b
a
where b is in a lower terms, i.e., (a, b) = 1. Then

a3
2= ⇐⇒ 2b3 = a3 .
b3

By the last a3 is even, that follows that a is even.

12
Then, for some k ∈ Z

a = 2k
3
=⇒ a = (2k)3 .

Then

2b3 = 8k 3
=⇒ b3 = 2(2k 3 ).

Then, b3 is even, consequently b is even. Then 2 is a common divisor of a and b,



then this is a contradiction. Then, 3 2 is irrational.

Problem 15. Let a and b be nonzero integers and let d = (a, b). Prove that a/d
and b/d are relatively prime.

Proof. Let d = a, b. To prove it let’s do it by Reductio ad absurdum.


Let’s suppose that
a b
( , = m) f or m > 1.
d d
Then
a
= u0 m and b = v0 m; u0 , v0 ∈ Z
d
=⇒ a = u0 (md) and b = v0 (md).

Then, (a, b) = md, where md > d, a contradiction. So then

a b
( , ) = 1.
d d

Problem 16. Let m, r, r′ ∈ Z. Prove that if (r, m) = 1 = (r′ , m), then (rr′ , m) = 1.

Proof. Let (r, m) = 1, then

1 = x0 + y0 m; x0 , y0 ∈ Z.

Similarly, for (r′ , m) = 1, then

13
1 = x1 r′ + y1 m; x1 , y1 ∈ Z.

Then

1 = (x0 r + y0 m)(x1 r′ + y1 m)
= x0 x1 rr′ + x0 y1 rm + y0 x1 r′ m + y0 y1 m2
= (x0 x1 )rr′ + (x0 y1 r + y0 x1 r′ + y0 y1 m)m.

Then
(rr′ , m) | [(x0 x1 )rr′ + (x0 y1 r + y0 x1 r′ + y0 y1 m)m] = 1,

and consequently
(rr′ , m) = 1.

Problem 17. Assume that d = sa + tb is a Z-linear combination of integers a and


b. Find infinitely many pairs of integers (sk , tk ) with d = sk a + tk b

Proof. Let d = sa + tb.


Then ∀k ∈ Z, we have that

d = sa + tb − kab + kab
= (s − kb)a + (t + ka)b.

Then (sk , tk ) = (s − kb, t + ka) is a solution for d. Then

{(sk , tk ) | k ∈ Z},

is a infinite set of solutions for d.

14
Problem 18. If a and b are relatively prime and if each divides an integer n, then
their product ab also divides n.

Proof. Let a, b ∈ Z such that (a, b) = 1.


If a|b and b|n then

n = x0 a and n = y0 b; x0 , y0 ∈ Z.

Then
x0 a = y0 b

=⇒ b|x0 b.

Since (a, b) = 1, then we obtain


b|x0 .

Then
x0 = m0 b, m0 ∈ Z.

Finally
n = x0 a = m0 ba

=⇒ ba|n.

Problem 19. Let a, b, c ∈ Z with a > 0. Prove that a(b, c) = (ab, ac). (One must
assume that a > 0 lest a(b, c) be negative.)

Proof. Let a, b, c ∈ Z, a > 0.


Let d = (b, c) and D = (ab, ac).
As a linear combination of d, we obtain the following result

d = x0 b + x1 c =⇒ ad = x0 ab + x1 ac, f or x0 , x1 ∈ Z. (19)

As a linear combination of D, we get that

D = y0 ab + y1 ac, f or y0 , y1 ∈ Z. (20)

From d = (b, c) we get that

d|c ∧ d|b =⇒ dk0 = c ∧ dk1 = b, f or k0 , k1 ∈ Z.

15
Then

adk0 = ac ∧ adk1 = ab; k0 , k1 ∈ Z


=⇒ ad|ac ∧ ad|ab.

As ad divides any linear combination of ac and ab, then by (20), ad divides D, i.e.,
ad|D.
From D = (ab, ac) we get that

D|ab ∧ D|ac.

As D divides any linear combination of ab and ac, then D divides ad by (19).


As D|ad and ad|D, that follows that (ab, ac) = D = ad = a(b, c).

Problem 20. A Pythagorean triple is a 3-tuple (a, b, c) of positive integers for


which

a2 + b2 = c2 .

A Pythagorean triple is called primitive if gcd(a, b, c) = 1. (Definition. A


common divisor of nonzero integers a1 , a2 , . . . , an is an integer c such that c | ai for
all i ∈ {1, . . . , n}. The largest of the common divisors is called its greatest common
divisor.)
(a) Consider a complex number z = q + ip, where q > p are positive integers.
Prove that

q 2 − p2 , 2qp, q 2 + p2


is a Pythagorean triple by showing that z 2 = |z|2 . (One can prove that every
primitive Pythagorean triple (a, b, c) is of this type.)

Proof. First, we compute the modulus squared of z:

|z|2 = (q + ip)(q − ip) = q 2 − (ip)2 = q 2 + p2 .

16
Next, we square z:

z 2 = (q + ip)2 = q 2 + 2qip − p2 .

The modulus squared of z 2 is:

|z 2 |2 = (q 2 − p2 )2 + (2qp)2 .

By equating |z 2 |2 and |z|4 , we find that:

(q 2 − p2 )2 + (2qp)2 = (q 2 + p2 )2 ,

which is in the form of a2 + b2 = c2 . Thus, (a, b, c) = (q 2 − p2 , 2qp, q 2 + p2 ) is a


Pythagorean triple.
Furthermore, if q and p are coprime and not both odd, then (a, b, c) forms a
primitive Pythagorean triple, since gcd(a, b, c) = 1. This concludes the proof that
every primitive Pythagorean triple can be represented by (q 2 − p2 , 2qp, q 2 + p2 ) for
some coprime positive integers q > p, where not both are odd.

(b) Show that the Pythagorean triple (9, 12, 15) (which is not primitive) is not
of the type given in part (a).

Proof. Suppose there are q and p for (9, 12, 15). Then 2qp = 12 and qp = 6. Since
q > p are positive integers, the only possibilities are q = 6 and p = 1 or q = 3
and p = 2. The first possibility gives the Pythagorean triple (12, 35, 37) while the
second gives the Pythagorean triple (5, 12, 13).

Problem 21. Let X and Y be finite sets. Show that there is a bijection f : X → Y
if and only if |X| = |Y |. (By definition, a set is finite if it is empty or if it can
be put into a one-to-one correspondence with [k] = {1, 2, . . . , k}, for some integer
k ≥ 1.

Proof. To prove this theorem, we show both implications:


(⇒) Assume there is a bijection f : X → Y . Since f is injective, each element
of X maps to a unique element of Y , thus |X| ≤ |Y |. Since f is also surjective,
every element of Y has a preimage in X, therefore |Y | ≤ |X|. Hence, |X| = |Y |.
(⇐) Now assume |X| = |Y | = k, for some nonnegative integer k. Enumerate
the elements of X as X = {x1 , x2 , . . . , xk } and those of Y as Y = {y1 , y2 , . . . , yk }.

17
Define a function f : X → Y by setting f (xi ) = yi for each i = 1, 2, . . . , k. This
function is clearly injective and surjective, hence bijective.
Thus, a bijection exists between X and Y if and only if |X| = |Y |, completing
the proof.

Problem 22. (Pigeonhole Principle) If X and Y are finite sets with the same
number of elements, show that the following conditions are equivalent for a function
f :X →Y.

(a) f is bijective

(b) f is injective

(c) f is surjective

Proof. We shall prove each condition implies the other two for a function f : X → Y ,
given that |X| = |Y |:
Bijective ⇒ Injective and Surjective: By definition, a bijective function is
one that is both injective (one-to-one) and surjective (onto).
Injective ⇒ Bijective: Assume f is injective. Since |X| = |Y |, for each y ∈ Y ,
there must be a distinct x ∈ X such that f (x) = y, otherwise, we would have fewer
images in Y than elements in X. As f covers all elements in Y and maps distinctly,
f is also surjective, and thus bijective.
Surjective ⇒ Bijective: Now assume f is surjective. Given that |X| = |Y |,
every element y ∈ Y is the image of at least one x ∈ X. Since there cannot be more
images in Y than elements in X, f must map distinct elements of X to distinct
elements of Y , which means f is injective. Hence, f is bijective.
Therefore, we have shown that for finite sets X and Y with equal cardinality,
being injective, surjective, and bijective are equivalent conditions for a function
f :X →Y.

Problem 23.

(a) Let f : X → Y be a function, and let (Si )i∈I be a family of subsets of X.


S  S
Prove that f i∈I Si = i∈I f (Si )

S  S
Proof. To prove that f i∈I Si = i∈I f (Si ), we will show that each side
is a subset of the other.
S  S
Step 1: f i∈I Si ⊆ i∈I f (Si )

18
S 
Take any y ∈ f i∈I Si . By the definition of the image under f , there
S S
exists at least one x ∈ i∈I Si such that f (x) = y. Since x ∈ i∈I Si , there
exists some index j ∈ I such that x ∈ Sj . Applying f to this relationship,
S
we obtain y = f (x) ∈ f (Sj ). Therefore, y ∈ i∈I f (Si ), showing that
S  S
f i∈I Si ⊆ i∈I f (Si ).
S S 
Step 2: i∈I f (Si ) ⊆ f i∈I Si
S
Now take any y ∈ i∈I f (Si ). This means there exists an j ∈ I such that
y ∈ f (Sj ). By the definition of the image, for this j, there is an x ∈ Sj such
S S
that f (x) = y. Since Sj ⊆ i∈I Si , we also have x ∈ i∈I Si , and therefore
S  S S 
y = f (x) ∈ f i∈I Si . This shows that i∈I f (Si ) ⊆ f i∈I Si .
S  S
Having shown both inclusions, we conclude that f i∈I Si = i∈I f (Si ),
completing the proof.

(b) If S1 and S2 are subsets of a set X, and if f : X → Y is any function, prove


that f (S1 ∩ S2 ) ⊆ f (S1 ) ∩ f (S2 ). Give an example in which f (S1 ∩ S2 ) ̸=
f (S1 ) ∩ f (S2 ).

Proof. Take any element y in f (S1 ∩ S2 ). This means there exists at least
one element x in S1 ∩ S2 such that f (x) = y. Since x is in S1 ∩ S2 , it must
be in both S1 and S2 . Applying f , we have that f (x) is in both f (S1 ) and
f (S2 ). Therefore, y is in f (S1 ) ∩ f (S2 ), which shows that f (S1 ∩ S2 ) ⊆
f (S1 ) ∩ f (S2 ).

Example 1. Consider the sets X = {a, b, c} and Y = {1, 2}. Let S1 = {a, b}
and S2 = {b, c}. Define a function f as follows:

f (a) = 1, f (b) = 2, f (c) = 1

Here, S1 ∩ S2 = {b}, hence f (S1 ∩ S2 ) = {2}.


However, f (S1 ) = {1, 2} and f (S2 ) = {1, 2}, so f (S1 ) ∩ f (S2 ) = {1, 2}.
We observe that f (S1 ∩ S2 ) = {2} =
̸ {1, 2} = f (S1 ) ∩ f (S2 ), which provides
the required example where the sets are not equal.

(c) If S1 and S2 are subsets of a set X, and if f : X → Y is an injection, prove


that f (S1 ∩ S2 ) = f (S1 ) ∩ f (S2 ).

Proof. To show that f (S1 ∩ S2 ) = f (S1 ) ∩ f (S2 ), we need to prove two


inclusions: f (S1 ∩ S2 ) ⊆ f (S1 ) ∩ f (S2 ) and f (S1 ) ∩ f (S2 ) ⊆ f (S1 ∩ S2 ).

19
First inclusion: This part is straightforward and does not require injectivity.
Take any element y in f (S1 ∩ S2 ). By the definition of the image, there exists
an element x in S1 ∩ S2 such that f (x) = y. Since x is in both S1 and S2 , it
follows that y ∈ f (S1 ) and y ∈ f (S2 ), hence y ∈ f (S1 ) ∩ f (S2 ). This shows
that f (S1 ∩ S2 ) ⊆ f (S1 ) ∩ f (S2 ).
Second inclusion: For the reverse inclusion, assume y ∈ f (S1 ) ∩ f (S2 ).
Then there exist x1 ∈ S1 and x2 ∈ S2 such that f (x1 ) = y and f (x2 ) = y.
Since f is injective, the fact that f (x1 ) = f (x2 ) implies x1 = x2 . Therefore,
x1 (which is equal to x2 ) is in both S1 and S2 , or in other words, x1 ∈ S1 ∩ S2 .
Hence, y = f (x1 ) ∈ f (S1 ∩ S2 ), which shows that f (S1 ) ∩ f (S2 ) ⊆ f (S1 ∩ S2 ).
Since both inclusions have been shown, we conclude that f (S1 ∩ S2 ) =
f (S1 ) ∩ f (S2 ).

Problem 24. Let f : X → Y be a function.

(a) If (Bλ )λ∈Λ is a family of subsets of Y , prove that


! !
[ [ \ \
−1 −1 −1
f Bλ = f (Bλ ) and f Bλ = f −1 (Bλ ).
λ∈Λ λ∈Λ λ∈Λ λ∈Λ

Proof. To prove the first equality:


Take any x ∈ f −1
S  S
λ∈Λ Bλ . This means f (x) ∈ λ∈Λ Bλ , so there exists
a λ ∈ Λ such that f (x) ∈ Bλ . Consequently, x ∈ f −1 (Bλ ), and therefore
x ∈ λ∈Λ f −1 (Bλ ). This shows f −1 −1
S S  S
λ∈Λ Bλ ⊆ λ∈Λ f (Bλ ).
Conversely, take any x ∈ λ∈Λ f −1 (Bλ ). This means there exists a λ ∈ Λ for
S

which x ∈ f −1 (Bλ ), implying f (x) ∈ Bλ and thus f (x) ∈ λ∈Λ Bλ . Hence,


S

x ∈ f −1 −1
(Bλ ) ⊆ f −1
S  S S 
λ∈Λ Bλ , showing λ∈Λ f λ∈Λ Bλ .

To prove the second equality:


Take any x ∈ f −1
T  T
λ∈Λ Bλ . This means f (x) ∈ λ∈Λ Bλ , so for all λ ∈ Λ,
f (x) ∈ Bλ . Consequently, x ∈ f −1 (Bλ ) for all λ, and thus x ∈ λ∈Λ f −1 (Bλ ),
T

proving f −1 −1
T  T
λ∈Λ Bλ ⊆ λ∈Λ f (Bλ ).
Conversely, take any x ∈ λ∈Λ f −1 (Bλ ). This means for all λ ∈ Λ, x ∈
T

f −1 (Bλ ), implying f (x) ∈ Bλ for all λ. Therefore, f (x) ∈ λ∈Λ Bλ , and


T

hence x ∈ f −1 −1
(Bλ ) ⊆ f −1
T  T T 
λ∈Λ Bλ , showing λ∈Λ f λ∈Λ Bλ .

Thus, we have shown both equalities, completing the proof.

(b) If B ⊆ Y , prove that f −1 (B c ) = f −1 (B)c , where B c denotes the complement


of B respect to Y .

20
Proof. To show that f −1 (B c ) = f −1 (B)c , we will prove two inclusions:
f −1 (B c ) ⊆ f −1 (B)c and f −1 (B)c ⊆ f −1 (B c ).
First inclusion: Assume x ∈ f −1 (B c ). This means that f (x) ∈ B c , or in
other words, f (x) ∈ / f −1 (B), and so x ∈ f −1 (B)c . Hence,
/ B. Therefore, x ∈
we have f −1 (B c ) ⊆ f −1 (B)c .
Second inclusion: Now, assume x ∈ f −1 (B)c . This means x ∈ / f −1 (B), or
/ B. Therefore, f (x) ∈ B c , implying x ∈ f −1 (B c ). Thus,
equivalently, f (x) ∈
we have f (B) ⊆ f −1 (B c ).
−1 c

Since we have shown both inclusions, it follows that f −1 (B c ) = f −1 (B)c ,


completing the proof.

Problem 25. Let f : X → Y be a function. Define a relation on X by x ∼ x′ if


f (x) = f (x′ ). Prove that ∼ is an equivalence relation. (If x ∈ X and f (x) = y, the
equivalence class [x] is usually denoted by f −1 ({y}), the inverse image of {y}.)

Proof. To prove that ∼ is an equivalence relation, we must show that it satisfies


reflexivity, symmetry, and transitivity.
Reflexivity: For any x ∈ X, we have f (x) = f (x), which means x ∼ x. Hence,
the relation is reflexive.
Symmetry: Assume x ∼ x′ for some x, x′ ∈ X. This means f (x) = f (x′ ). By
the definition of equality, f (x′ ) = f (x) as well, and thus x′ ∼ x. Therefore, the
relation is symmetric.
Transitivity: Assume x ∼ x′ and x′ ∼ x′′ for some x, x′ , x′′ ∈ X. This
implies f (x) = f (x′ ) and f (x′ ) = f (x′′ ). By the transitive property of equality,
f (x) = f (x′′ ), which means x ∼ x′′ . Hence, the relation is transitive.
Since ∼ satisfies reflexivity, symmetry, and transitivity, it is an equivalence
relation.

21

You might also like